Last visit was: 29 Apr 2024, 06:24 It is currently 29 Apr 2024, 06:24

Close
GMAT Club Daily Prep
Thank you for using the timer - this advanced tool can estimate your performance and suggest more practice questions. We have subscribed you to Daily Prep Questions via email.

Customized
for You

we will pick new questions that match your level based on your Timer History

Track
Your Progress

every week, we’ll send you an estimated GMAT score based on your performance

Practice
Pays

we will pick new questions that match your level based on your Timer History
Not interested in getting valuable practice questions and articles delivered to your email? No problem, unsubscribe here.
Close
Request Expert Reply
Confirm Cancel
Tags:
Show Tags
Hide Tags
GRE Forum Moderator
Joined: 02 Nov 2016
Posts: 13966
Own Kudos [?]: 33035 [1]
Given Kudos: 5781
GPA: 3.62
Send PM
Intern
Intern
Joined: 23 Jun 2021
Posts: 47
Own Kudos [?]: 29 [0]
Given Kudos: 245
Location: India
GPA: 3.54
Send PM
Intern
Intern
Joined: 05 Jan 2024
Posts: 1
Own Kudos [?]: 0 [0]
Given Kudos: 2
Location: India
Send PM
GRE Forum Moderator
Joined: 02 Nov 2016
Posts: 13966
Own Kudos [?]: 33035 [0]
Given Kudos: 5781
GPA: 3.62
Send PM
Re: For the past few years, the per store sales of a drugstore chain have [#permalink]
Expert Reply
Official Explanation

The satisfaction of customers who shop at the company’s stores has improved, and the company is advertising this fact. Choice (D) provides evidence that these moves are having a positive effect, and suggests that they may result in higher sales: If people who do not shop at this company’s stores think better of the company now than they used to, then there is a possibility that they will be converted to customers of the company. There is no guarantee of this, of course, but there is no other statement that provides stronger support for the argument, so choice (D) is the correct answer in the first column.

Choice (A) seems as though it might weaken the argument, but in fact it does not provide new meaningful information one way or the other: whether or not the previous management team was focused on customer satisfaction, the results are what matters.

Choice (C) basically says that the survey results are statistically valid, but it does not give any further evidence that these results will or will not lead to the conclusion presented in the argument. (If, instead, the statement had said that the survey results are not statistically valid, then it would have been a good candidate for the statement that undermines the conclusion.)

Choice (E) seems tempting, but is ultimately irrelevant: The conclusion says that the company will increase its per store sales, not its overall sales. You have no information on which to judge how the new stores will affect the company’s per store sales. Finally, choice (B) is the one that undermines the argument.

The correct answer is (D).

The conclusion that the company’s per store sales will increase rests partly on the improved customer satisfaction results. Choice (B) says that only a score of 5 in customer satisfaction correlates strongly with increased sales. Therefore, if most of the 75% of respondents who rated their satisfaction a 4 or a 5 in fact rated it a 4, then the force or the survey results, in terms of predicting increased sales, becomes much weaker. [While choice (B) does not tell you what the breakdown of 4s and 5s is, it does give reason to doubt the significance of that 75% figure.]

The correct answer is (B).
GMAT Club Bot
Re: For the past few years, the per store sales of a drugstore chain have [#permalink]
Moderators:
Math Expert
92990 posts
DI Forum Moderator
1030 posts
RC & DI Moderator
11185 posts

Powered by phpBB © phpBB Group | Emoji artwork provided by EmojiOne